ChaseDream

标题: 请看GWD-5-15 [打印本页]

作者: linxi8389    时间: 2004-10-5 23:09
标题: 请看GWD-5-15

Q15:


Lightbox, Inc., owns almost all of the movie theaters in Washington County and has announced plans to double the number of movie screens it has in the county within five years.  Yet attendance at Lightbox’s theaters is only just large enough for profitability now and the county’s population is not expected to increase over the next ten years.  Clearly, therefore, if there is indeed no increase in population, Lightbox’s new screens are unlikely to prove profitable.






Which of the following, if true about Washington County, most seriously weakens the argument?






  1. Though little change in the size of the population is expected, a pronounced shift toward a younger, more affluent, and more entertainment-oriented population is expected to occur.

  2. The sales of snacks and drinks in its movie theaters account for more of Lightbox’s profits than ticket sales do.

  3. In selecting the mix of movies shown at its theaters, Lightbox’s policy is to avoid those that appeal to only a small segment of the movie going population.

  4. Spending on video purchases, as well as spending on video rentals, is currently no longer increasing.

  5. There are no population centers in the county that are not already served by at least one of the movie theaters that Lightbox owns and operates.

The key is A, I chose C. C is also logic, since the company can also increase the population who come to see the cinema.Please give me some tips. Thank you!



作者: agnesqin    时间: 2004-10-8 11:40
C.这个行为可能之前就已经这样做了。所以不能表示增加之后能够带来额外的收益。
作者: plaster    时间: 2004-10-13 15:08
C is out of the scope,文中没有提到怎么选电影,如果要weaken evidences即人口十年之内不会增加, A中阐明了潜在人口的增加,起到了weaken的作用. 呵呵..其实我开始和你犯了一样的错误.
作者: dinosaure    时间: 2004-11-13 17:54
支持C; A中所谓的a younger, more affluent, and more entertainment-oriented population 并不能直接转换为会多看电影的population.而C中 is to avoid those that appeal to only a small segment of the movie going population就指明了选取放映影片时要努力吸引不同层面的观众,但如果is 改为will就更好;比较起来,A更不合理,选C
作者: dinosaure    时间: 2004-11-14 13:01
up
作者: cherylcui_mba    时间: 2004-11-23 21:09

和四楼的理解一样.觉得C更合理.

请问NN们的理解?


作者: aokceo    时间: 2004-11-24 14:42
why not choose the answer B ? personally think neither A nor C is correct . because the A just show a trend ,and C tell us a strategy the company will take , rather than a direct force to the population . it is clearly , the problem is why no increase in the population will not definitely led to the decrease of profit , because the B demonstrate that the profit did not come from the revenue by sale the ticket to the population . that is to say , there is a possibility that rise the price of drink or by other measures to earn more profit  without obviously increase the population . so how do you think ?
作者: tempture    时间: 2004-12-7 11:08
题目是profit的变化,选项也就是要有比较才对,BC都没有比较,A不是好选项,但是最靠谱。
作者: roland1972    时间: 2004-12-7 13:09
支持b
作者: yyd9806    时间: 2004-12-18 08:01

支持B. 文中没有表明A与C和利润有关系. 尽管A与C能增加看电影的人数, 但看电影的人数增加就一定能增加利润吗? 例如, 每卖一张电影票就亏5美分, 卖的越多亏的越多. 如果利润主要是电影票以外的因素产生的,例如食品; 人数的增加促进了食品的销售从而提高了利润, 就如B所说的, 能够起到削弱作用....


作者: Mandyxie    时间: 2004-12-23 12:54
支持C, A必须有一个假设在先:more entertainment-oriented population 必须喜欢看电影,而不是别的方式,而本身这个假设就是有问题的,而只有C最直接地反映了在总人数不增加的情况下增加看电影的人数。。而B is out of scope..
作者: xionghuixh    时间: 2005-1-18 13:24

没有牛人发表意见吗???


作者: jd-benyou    时间: 2005-1-24 11:51

大家应该还记得有一题是,超市选择低卡果酱和通常果酱的问题吧,我觉得这是个如何增加量的问题,从这个角度看,c防止了原有顾客流失的情况下拓宽客户面的问题,通过同时播放多部影片满足不同客户群的口味增加观众,当然需要更多的播放厅啦,想想现在的高档电影厅的做法,就比较容易理解c是最好的解释。

就是如何diversify客户群的问题啊,而原先的播放厅不够多,所以同时播放的片子类型少观众就少,而增加播放厅的话就可以同时播放不同的片子吸引不同的观众,即使从字面上看a也是没有贴近原文的:年轻的爱娱乐的就一定多看电影吗?这里面需要进一步的假设。


作者: Ggirl    时间: 2005-1-27 14:34
投A一票
作者: 妞妞的牛牛    时间: 2005-3-31 11:10
以下是引用sherry89550473在2005-3-25 16:04:00的发言:

雖然B和C都能使戲院收益增加~


但是


都不符合題目的焦點!


題目的焦點是: 看電影"人口"的問題!


因此我贊成A!


(我一開始選C但後來覺得A較合理)


同意sherry的,题目主要关注人口不变的情况


作者: scorpio0001    时间: 2005-4-15 15:18

支持A。

C的错误:

1、本文只讨论增加银幕的数量和观众数量的问题,并没有论及如何选电影。

2、C中的方法在没有增加银幕数量的情况下也能做,所以与措施无关。


作者: ethyl    时间: 2005-5-17 23:43
注意结论:

Clearly, therefore, if there is indeed no increase in population, Lightbox’s new screens are unlikely to prove profitable.


A:虽然人口没有增加,但电影消费群扩大了,断桥削弱


B:sales of snacks and drinks,典型无关,讨论的是荧幕的盈利可能性


C:selecting the mix of movies,所犯错误同B,无关


D:Spending on video purchases, as well as spending on video rentals,同BC,继续无关


E:所有地区都有L控制的影院,加强



可见,ETS的伎俩并不高明,同一个手法在一道题里面竟然上演三次,简直可恶……


[此贴子已经被作者于2005-5-17 23:44:23编辑过]

作者: xinfaxian    时间: 2005-6-2 12:28

B够干扰的,想了半天才明白:就算profit的构成是象B所说的,但前提还是人群要增加,不增加的话,卖饮料卖的再好也没有用。ETS还是很阴险的啊,干扰偶们的视线!


作者: roberto0220    时间: 2005-6-27 20:17

支持c。c的意思是说电影选择政策是避免那些只吸引看电影的人中的一小部分人的电影Lightbox’s policy is to avoid those that appeal to only a small segment of the movie going population),也就是说看电影的人总量也许不会变,但那些人完全可能会看了电影a又看电影b----因为a和b都会吸引他们----政策导致的。开多电影厅当然likely  to profit。是不是削弱?a的问题在于a还需要进一步假设。


To be discussed



作者: stellaecon    时间: 2005-7-1 21:41
同意ethyl说的。而且这道题问的是“if true about Washington County”,那那个电影院的政策并不和Washington County直接相关吧。不一定就这样否定了,但是看C就是不顺
作者: joeysue    时间: 2005-7-16 13:08

a~


[此贴子已经被作者于2005-7-16 13:11:05编辑过]

作者: beautyup    时间: 2005-8-4 15:12

可以从时态上甄别一下:


a 说的是将来时,虽然人口不增加,但是消费者群增加了。更加针对原文的推理。


c 可能说的现在已经采取这样的策略了。


作者: suxifan    时间: 2005-8-5 01:13
"is to avoid" indicates the policy can be impending.


作者: joypainter    时间: 2005-8-7 14:59
GWD5-1. AC有争议。不过,题目是问关于new screens的问题,主要是新增的屏幕有没有人来看。按A,总的人数会更多,新增屏幕可能会有人来看。按C,影院的电影品种减少了。C若成立,暗含假设是原来那些热门品种的电影是供不应求的(所以才可以通过分配更多的时间/屏幕给热门电影来增加利润),但这一假设并未明示,所以C只能是无关选项。
作者: dongyuntao    时间: 2005-8-8 21:42

cn贴啊,不好意思了...


偶觉得B不对的原因有两个


首先怀疑看电影人数增加利润就增加这件事太没有道理了,你看原文都说。yet... and and the county’s population is not expected to increase over the next ten years.也就是文章都认为这个假设是成立的,所以它才说这句否定人口增加带来利润增加这种可能


其次,如果B正确,那么与人口就没有什么关系了,既然人数都没有变化,那么也就谈不上零食销售的利润了,这样B反而成了加强选项。


作者: sensornet    时间: 2005-8-14 17:27

同意A,因为题设问的是if true about Washington Country, most seriously weakens the agrument。如果问的是,if true about Lightbox Inc., most seriously weakens the argument,那么肯定选择C。


单纯问削弱作用,C显然强于A,因为A 中需要进一步说明enrerainment oriented 和 watching movie的关系。但是A最符合原题的问题,所以选A。试想如果考试的时候,ETS把问题改称我说的第二种问法,大家选什么??


作者: stray2000    时间: 2005-9-25 01:52

偶不是NN,但是这道题目当时一看就选了A。


个人认为B,C选项都已经包含在原文


Yet attendance at Lightbox’s theaters is only just large enough for profitability now


这句话的范围内。也就是说 attendance 已经考虑到B(小吃利润高于售票)和C(挑选影片组合避免不够吸引力)。


或者说,BC都是现在状态,对未来没有影响。


另外,结论中为何多提一句, if there is indeed no increase in population,这个和前面and the county’s population is not expected to increase over the next ten years. 明显是同义。所以,考察重点肯定是population的变化


作者: 我爱欧洲    时间: 2005-10-13 16:32
题目说增加"电影"的利润,B项说的是零食和饮料,无关.
作者: joanrain    时间: 2005-10-23 06:19

我选的也是c,a更像是无关选项


那些人群不一定就喜欢看电影啊


比较起来,c选项指出可以吸引更多的看电影人群所以不会导致less profitable!~~



作者: zhoujian    时间: 2005-12-11 16:45
以下是引用ethyl在2005-5-17 23:43:00的发言:
注意结论:

Clearly, therefore, if there is indeed no increase in population, Lightbox’s new screens are unlikely to prove profitable.


A:虽然人口没有增加,但电影消费群扩大了,断桥削弱


B:sales of snacks and drinks,典型无关,讨论的是荧幕的盈利可能性


C:selecting the mix of movies,所犯错误同B,无关


D:Spending on video purchases, as well as spending on video rentals,同BC,继续无关


E:所有地区都有L控制的影院,加强


可见,ETS的伎俩并不高明,同一个手法在一道题里面竟然上演三次,简直可恶……



同意,我觉得做这种题目还是要跟着ETS的思路走,不然搞不好就要out of scope


作者: steveyangxt    时间: 2005-12-12 11:16
以下是引用dinosaure在2004-11-13 17:54:00的发言:
支持C; A中所谓的a younger, more affluent, and more entertainment-oriented population 并不能直接转换为会多看电影的population.而C中 is to avoid those that appeal to only a small segment of the movie going population就指明了选取放映影片时要努力吸引不同层面的观众,但如果is 改为will就更好;比较起来,A更不合理,选C

说的好 c的问题就在这里。用is表示现在都采用了, 要增长也是现在增长, 增长过了。 表示出了人口增长这个因素,年轻人的增长也可以 理解为相对人口增长嘛


作者: mko    时间: 2006-2-1 15:35
B再加一句,从增加screen后,突然之间看电影的人都买双份的snacks 和drinks...这样就有点靠谱了.

作者: jinni    时间: 2006-3-2 12:51

这道题的结论是带条件型的结论,Clearly, therefore, if there is indeed no increase in population, Lightbox’s new screens are unlikely to prove profitable.


对于这种形式的weaken题,选项必然集中在if所说的问题上。或举反例子,或提供有关信息。


这里说即使没有人口的数量上的增长,但是在人口中喜欢娱乐消费的人比例增加,所以利润是可以增加的。



[此贴子已经被作者于2006-3-2 12:59:34编辑过]

作者: whydog    时间: 2006-8-10 23:14
以下是引用sensornet在2005-8-14 17:27:00的发言:

同意A,因为题设问的是if true about Washington Country, most seriously weakens the agrument。如果问的是,if true about Lightbox Inc., most seriously weakens the argument,那么肯定选择C。

单纯问削弱作用,C显然强于A,因为A 中需要进一步说明enrerainment oriented 和 watching movie的关系。但是A最符合原题的问题,所以选A。试想如果考试的时候,ETS把问题改称我说的第二种问法,大家选什么??

我同意这个,其实题目的关键在于问题这句话中的if true about Washington Country.A和C都是可以扩大看电影的人数的.如果ETS把问题改成第二种问法,那么就必然是C了


作者: ecochem    时间: 2006-9-23 01:05
以下是引用jinni在2006-3-2 12:51:00的发言:

这道题的结论是带条件型的结论,Clearly, therefore,
                if there is indeed no increase in population,
Lightbox’s new screens are unlikely to prove profitable.

对于这种形式的weaken题,选项必然集中在if所说的问题上。或举反例子,或提供有关信息。

这里说即使没有人口的数量上的增长,但是在人口中喜欢娱乐消费的人比例增加,所以利润是可以增加的。



说得真好!
作者: 想桐桐的蝎子    时间: 2006-9-28 20:03

  C的错误:

1、本文只讨论增加银幕的数量和观众数量的问题,并没有论及如何选电影。

2C中的方法在没有增加银幕数量的情况下也能做,所以与措施无关。C.这个行为可能之前就已经这样做了。所以不能表示增加之后能够带来额外的收益。

年轻的爱娱乐的就一定多看电影吗?这里面需要进一步的假设。所以A的确出得不好,但是就题目的focus来说,讨论的是未来,思来想去还是A吧,題目的焦點是: 看電影"人口"的問題

在NN的基础上总结一下我认为NN们比较好的解释


作者: cattsky    时间: 2006-10-3 20:54

支持A,典型的需求量问题

人口衰退VS消费能力;数量VS效率

数量下降等于总需求量下降吗?效率增加就可以进行削弱.

GWD里有好几道这样的例子

此文的key在于人口停滞和消费增长,文章的逻辑是:no population increase--no more people to cinema--no more profit

其中假设是 人均消费量不变, A正好削弱了这个假设.

假如如B所说,profit主要是由小吃的销售组成的;但这能改变去影院的人数,能促进人均销售量吗?所以无法进行削弱


[此贴子已经被作者于2006-10-3 21:02:22编辑过]

作者: fiftycents86    时间: 2006-10-3 22:56
B肯定不对,你看题干是对Washington County哪个起到weaken作用,而不是theater本身
作者: luckyouyang    时间: 2006-11-29 05:16

想来想去还是C,经常去看电影的基本都知道,很多时候喜欢看的电影没有合适的时间安排,要不就是好的位置已经没了。因此,增加银幕,可以增加挑选的余地,刺激了电影票的销售。

to avoid those that appeal to only a small segment of the moviegoing population.

A是说不过去的,因为一般来说更年轻,更富裕,更娱乐的人群,选择的娱乐方式更多,未必会去看电影。

举个例子,中国电影业票房的高峰期在80年代,明显不是现在。尽管人口更多了,人们更富裕了,年轻人也更多了。


作者: ana9    时间: 2006-11-29 13:01
agree~~~
作者: dreamalittle    时间: 2007-2-15 18:30

作者: dreamalittle    时间: 2007-2-15 18:32
以下是引用stray2000在2005-9-25 1:52:00的发言:

偶不是NN,但是这道题目当时一看就选了A。

个人认为B,C选项都已经包含在原文

Yet attendance at Lightbox’s theaters is only just large enough for profitability now
        

这句话的范围内。也就是说 attendance 已经考虑到B(小吃利润高于售票)和C(挑选影片组合避免不够吸引力)。

或者说,BC都是现在状态,对未来没有影响。

另外,结论中为何多提一句, if there is indeed no increase in population,这个和前面and the county’s population is not expected to increase over the next ten years. 明显是同义。所以,考察重点肯定是population的变化

牛,同意!
作者: Sacramento_CA    时间: 2007-6-2 10:32

昨天review这题,又错了!

现在看起来,这是属于“条件”类的weaken题目。原文:Clearly, therefore, if there is indeed no increase in population, Lightbox’s new screens are unlikely to prove profitable.  也就是A--->B的推理,其中A是(充分条件):there is indeed no increase in population,B(必要条件)是:Lightbox’s new screens are unlikely to prove profitable。

此类题目上XDF的时候老师较的做法是:一看到条件类推理的词,如这里的IF,就攻击必要条件,也就是说:原文说人口没增长会导致L不可能提高利润,攻击的时候就要说:即使人口减少,L也可以增加利润。看看A选项是如此完美的对应:Though little change in the size of the population is expected(充分条件是一样的),a pronounced shift toward a younger, more affluent, and more entertainment-oriented population is expected to occur.(必要条件被攻击了)。

觉得这方法满不错的,但是做题的时候总是没有第一时间反应过来。说白了还是功底不够。


作者: нандин    时间: 2007-8-13 21:04
以下是引用Sacramento_CA在2007-6-2 10:32:00的发言:

昨天review这题,又错了!

现在看起来,这是属于“条件”类的weaken题目。原文:Clearly, therefore, if there is indeed no increase in population, Lightbox’s new screens are unlikely to prove profitable.  也就是A--->B的推理,其中A是(充分条件):there is indeed no increase in population,B(必要条件)是:Lightbox’s new screens are unlikely to prove profitable。

此类题目上XDF的时候老师较的做法是:一看到条件类推理的词,如这里的IF,就攻击必要条件,也就是说:原文说人口没增长会导致L不可能提高利润,攻击的时候就要说:即使人口减少,L也可以增加利润。看看A选项是如此完美的对应:Though little change in the size of the population is expected(充分条件是一样的),a pronounced shift toward a younger, more affluent, and more entertainment-oriented population is expected to occur.(必要条件被攻击了)。

觉得这方法满不错的,但是做题的时候总是没有第一时间反应过来。说白了还是功底不够。

听君一席话,胜做cr100题


作者: songyeonpp    时间: 2007-8-15 14:51

我觉得A不对. 说如果增加屏幕数就会出现更年轻,更有活力,更热爱娱乐的人群.这里犯了个错误,就是目标人物写得太具体且狭窄,吸引的应该不光是更年轻更有活力的人.

c: 屏幕数量增加了,观众的自然选择就更多,由此避免了以前只吸引少部分人来看电影的现象.


作者: 龙飞凤舞    时间: 2007-11-8 07:54

我开始支持C的,后来看了半天,觉得A更好

有时不要死扣,因为确实从C角度来说也能profitable,

但是文章最后一句话:Clearly, therefore, if there is indeed no increase in population, Lightbox’s new screens are unlikely to prove profitable.

再看A:A. Though little change in the size of the population is expected, a pronounced shift
toward a younger, more affluent, and more entertainment-oriented population is
expected to occur.

A中的Though就是让步,顺着题目意思往下讲,提出另外一个可能性,从而削弱。

即使选A,都不能完全说C错


作者: aircavalry    时间: 2007-11-8 19:33
以下是引用jinni在2006-3-2 12:51:00的发言:

这道题的结论是带条件型的结论,Clearly, therefore,
                if there is indeed no increase in population,
Lightbox’s new screens are unlikely to prove profitable.

对于这种形式的weaken题,选项必然集中在if所说的问题上。或举反例子,或提供有关信息。

这里说即使没有人口的数量上的增长,但是在人口中喜欢娱乐消费的人比例增加,所以利润是可以增加的。



嗯,跟你想得一样,应该注意审题,从population的角度去weaken,支持选A


作者: whiteshadow    时间: 2007-11-27 01:03

作者在结论中强调了削弱的条件,而这个条件,上文有提到,找到理由,进行削弱。


作者: whiteshadow    时间: 2007-11-27 01:04
如果以后削弱题中,作者给出削弱的理由,特别是在结论里,那就应该攻击这个理由
作者: tigercaiqun    时间: 2008-3-15 11:15
44楼和jinni,就是一个字:好!
作者: rpch2004    时间: 2008-5-29 08:40
以下是引用Sacramento_CA在2007-6-2 10:32:00的发言:

昨天review这题,又错了!

现在看起来,这是属于“条件”类的weaken题目。原文:Clearly, therefore, if there is indeed no increase in population, Lightbox’s new screens are unlikely to prove profitable.  也就是A--->B的推理,其中A是(充分条件):there is indeed no increase in population,B(必要条件)是:Lightbox’s new screens are unlikely to prove profitable。

此类题目上XDF的时候老师较的做法是:一看到条件类推理的词,如这里的IF,就攻击必要条件,也就是说:原文说人口没增长会导致L不可能提高利润,攻击的时候就要说:即使人口减少,L也可以增加利润。看看A选项是如此完美的对应:Though little change in the size of the population is expected(充分条件是一样的),a pronounced shift toward a younger, more affluent, and more entertainment-oriented population is expected to occur.(必要条件被攻击了)。

觉得这方法满不错的,但是做题的时候总是没有第一时间反应过来。说白了还是功底不够。

攻击的时候应该说:即使人口没有减少,L也可以增加利润。


作者: latinjazz    时间: 2008-7-3 13:57
以下是引用xinfaxian在2005-6-2 12:28:00的发言:

B够干扰的,想了半天才明白:就算profit的构成是象B所说的,但前提还是人群要增加,不增加的话,卖饮料卖的再好也没有用。ETS还是很阴险的啊,干扰偶们的视线!

呵呵 我也被这个选项骗了


作者: amy1028    时间: 2008-10-25 17:09

我也是被B骗了...


作者: zhengjingzhe    时间: 2009-7-22 20:28
up
作者: lzj1209    时间: 2009-11-8 11:13
同是被B骗的人啊~~
还是A最有关~
作者: ving88    时间: 2010-3-17 23:34
B错在SCREEN上
 C错在时态上,C是一种惯例
作者: Feelalive    时间: 2013-6-23 16:18
谢谢谢谢
作者: DiSian    时间: 2015-8-28 15:59
这里是说电影院准备double the screens 但是因为attendence勉强盈利+population不变所以预测不会盈利
这里的assumption是attendence不会变
所以选项应该是可以改变attendence的
A 改变目标市场 B.D无关
C E 和人口有关 但是是现有情况 也就是说电影院已经是这样的了所以没有change不能选
作者: ansionjoe    时间: 2015-8-29 01:08
这个题目很明显的是A啊!
逻辑链:company double screens,but no increase of population.---->no increase profit
方法:只要找到一个方法证明,虽然人口不增长,但是这种方法也是可以的。
A显然就是,虽然总人口不变,但是喜欢看电影的人会增长!
C显然无关啊,不选择冷门电影~~跟人口增长有毛关系啊~~  无关选项  而且你也不知道他现在是不是这么做的  如果现在也是  那么即便以后这么做  也并没有什么卵用啊

作者: ansionjoe    时间: 2015-8-29 01:08
这个题目很明显的是A啊!
逻辑链:company double screens,but no increase of population.---->no increase profit
方法:只要找到一个方法证明,虽然人口不增长,但是这种方法也是可以的。
A显然就是,虽然总人口不变,但是喜欢看电影的人会增长!
C显然无关啊,不选择冷门电影~~跟人口增长有毛关系啊~~  无关选项  而且你也不知道他现在是不是这么做的  如果现在也是  那么即便以后这么做  也并没有什么卵用啊

作者: ymeng52000    时间: 2015-12-11 02:01
试着写一下思路
因为题目中的"population is not expected to increase over the nest ten years" 是指country's population。所以题目是个悖论,全国人口不增加不等于电影院消费人口不会增加。手段目的的削弱题

A: 根据原文,全国人口没有增加,但是电影院吸引了更多消费者
B:无关,因为只有来看电影的人会买snacks
C:无关,因为没有提到原文的手段(增加屏幕)

一开始觉得可以用其他手段直接否定结论应该也是对的,但是相比起来A 更符合出题者的意图。不过判断是不是out of scope真是很难啊!

作者: angelaunique123    时间: 2017-9-2 01:23
结论new screens are unlikely to prove profitable.
文章反复在谈的就是人口多少和屏幕变大是否盈利这两个事情之间的关系.
A.用新增屏幕播放年轻人喜欢的电影,这样就增加年轻人这个受众。
*C中的方法在没有增加银幕数量的情况下也能做,所以与措施无关*。
本文只讨论增加银幕的数量和观众数量的问题,并没有论及如何选电影。C如果增加新屏幕后切换为更符合大众口味的电影,那么能吸引大众。
A.用一半屏幕(旧屏幕)切换为更符合大众口味的电影,吸引1/2C量的大众。剩下一半屏幕播放年轻人喜欢的电影吸引年轻人,因为年轻人比大众更富裕,更倾向于去娱乐 (more affluent, and more entertainment-oriented) ,所以吸引的年轻人数量一定大于C中剩下1/2的大众数量。 所以A的总增加人数比C多。




欢迎光临 ChaseDream (https://forum.chasedream.com/) Powered by Discuz! X3.3